AAMC CBT7 and 7R OFFICIAL Q&A

This forum made possible through the generous support of SDN members, donors, and sponsors. Thank you.

Vihsadas

No summer
Moderator Emeritus
Lifetime Donor
10+ Year Member
5+ Year Member
15+ Year Member
Joined
Oct 17, 2007
Messages
5,474
Reaction score
56
This is the official Q&A thread for AAMC CBT7 and 7R.

Please post ONLY questions pertaining to AAMC CBT7 and 7R.
Out of respect for people who may not have completed the other exams, do not post questions or material from any other AAMC exam.

Please see this thread for the rules of order before you post.

Good luck on your MCAT!

Members don't see this ad.
 
Hi. Can someone please explain #24 in the PS please. The question is :
What is the value of c in Equation 2 as determined from the data in Table 1?
KLM - I also had issues with this question and would like to hear from you/others. When doing the problem, I knew the numbers did not divide evenly or cleanly. I thought it was likely zero order:

trial 4/trial 2 --> 4^x = 4/3.5
4^x = ~1 (if u check on a calculator it is 1.14)

so x should be zero

However, I noticed zero order was not a choice. There is no way it is 2nd, 3rd, or 4th order - that would mean 4^x should be approximately 16, 64, or 256 respectively. So I picked "first order"

I might have made a calculation error but I don't get how this is first order not zero order
 
KLM - I also had issues with this question and would like to hear from you/others. When doing the problem, I knew the numbers did not divide evenly or cleanly. I thought it was likely zero order:

trial 4/trial 2 --> 4^x = 4/3.5
4^x = ~1 (if u check on a calculator it is 1.14)

so x should be zero

However, I noticed zero order was not a choice. There is no way it is 2nd, 3rd, or 4th order - that would mean 4^x should be approximately 16, 64, or 256 respectively. So I picked "first order"

I might have made a calculation error but I don't get how this is first order not zero order

first of all, i think you can kind of 'cheat' by looking at the next question (25) that states "reaction is first order with respect to both acetone and hydronium ion". but lets assume for now that question wasn't there.

obviously the first step is to realize we are looking at the exponent for hydronium ion. which means we keep the other reactants the same, in this case, only acetone (bromine is irrelevant since it is zero order as stated in the passage).

the mcsteak you made was, you thought the rate law was determined by the different rate constants, which is incorrect. the rate constant is a CONSTANT, it will not change. look at the numbers - its averages around 3.8.

what you need to look at is the 5th column in the chart, which has -delta[Br2]/deltaT Ms-1. lets look at exp 2, 4, and 6 for example since they keep acetone constant and only change hydronium ion. between exp 2 and 6, the hydronium ion is doubled, so the disappearance of Br2 also doubles (2.85 vs 5.99 is close enough to 3 vs 6). looking at exp 6 and 4, again the hydronium ion is doubled, and the disappearance of Br2 also doubles (6 vs ~12).

so basically the exponent is 1 since they are directly proportional (doubling the [reactant] doubles the rate of reaction).

THE RATE CONSTANT IS NOT THE RATE OF THE REACTION
 
^thanks for the explanation. i get what you are saying about looking at the Br2 column instead, but i didn't realize that is what you were suppose to do. why is that we always compare the rate constant in almost every other rate of reaction problem then?
 
Members don't see this ad :)
the mcsteak you made was, you thought the rate law was determined by the different rate constants, which is incorrect. the rate constant is a CONSTANT, it will not change. look at the numbers - its averages around 3.8.

Lukkie thanks for the very clear explanation. That makes sense - I should read the graphs/columns more carefully (I didn't even catch my error when reviewing the exam, in part due to AAMC's poor explanation).

why is that we always compare the rate constant in almost every other rate of reaction problem then?
Klm - usually, we compare our reactant concentrations to the rate of the reaction. (In rate of reaction problems, you never compare to the rate constant- it's always the rate).

For example, if I say rate = k [A]^2 ^1
if you double the concentration of B from 2 to 4, the rate will double.

however, the rate constant will NOT change - it stays the same.

The trick on this problem was realizing NOT to use the column with "rate constant k" and instead using [change in Br/ change in t], which is the rate.
 
For the aamc 7r

#28 [physical sciences]
the molar absorptivity of bromine at 395nm is 198M-1cm-1 what is the absorbance at 395nm in experiment 1?

The equation is A = e c l

according to the table c = 4.14*10^-3
e is same.. 198M-1cm-1
and l is given as 1cm

so wouldnt the answer be B: .820?
answer key says c: 1.22

the explanation i have for it seems to be for a different version of this test or something?

also #60 what is the mole fraction of lead nitrate in 10%( wt-wt) aq solution?

assume 100g total
10g * 1/330g = .03 mole pb(no3)2
90g * 1/18g = 5 mol H20

so wouldnt mole fraction be .03/[.03+5] = .03/5.03 =~ .6 ?

the answer says .03
 
Last edited:
^ oooh I understand now! i was getting confused between the rate constant and the rate of reaction. thanks guys!
 
For the aamc 7r

#28 [physical sciences]
the molar absorptivity of bromine at 395nm is 198M-1cm-1 what is the absorbance at 395nm in experiment 1?

The equation is A = e c l

according to the table c = 4.14*10^-3
e is same.. 198M-1cm-1
and l is given as 1cm

so wouldnt the answer be B: .820?
answer key says c: 1.22

the explanation i have for it seems to be for a different version of this test or something?

Yes the answer is B: .820. Must be a mistake in the answer key.

also #60 what is the mole fraction of lead nitrate in 10%( wt-wt) aq solution?

assume 100g total
10g * 1/330g = .03 mole pb(no3)2
90g * 1/18g = 5 mol H20

so wouldnt mole fraction be .03/[.03+5] = .03/5.03 =~ .6 ?

the answer says .03

.03/5 ~ .006...
 
right.. thanks

everyone agrees that the verbal on this one was ridiculous (hard and boring)?

why isnt there a better curve on the verbal then?
 
right.. thanks

everyone agrees that the verbal on this one was ridiculous (hard and boring)?

why isnt there a better curve on the verbal then?

a few people said it was easy but i thought it was one of the harder ones.
 
omg i just took this (full length version), 15 ps 11 vr 15 bs, I thought the verbal was RIDICULOUS, I missed 12! but somehow that translates into an 11. Glad to see everyone else thinks the same way though, I was worried for a bit.
 
omg i just took this (full length version), 15 ps 11 vr 15 bs, I thought the verbal was RIDICULOUS, I missed 12! but somehow that translates into an 11. Glad to see everyone else thinks the same way though, I was worried for a bit.

Wow.
 
Members don't see this ad :)
Can someone plese help me out with these? The explanations are so general and i feel like these are just things we should know, rather then obtain from the passage...

#110 in BS: Normally the immune system avoides attacking the tissues of its own body because:

A) special intracellular process recognizes only foreign antigens
B) the body does not make any antigens that the immune system could recognize
C) It changes its antibodies to be specific only to foreign antigens
D) it suppresses cells specific to the body's own antigens

---The answer is D but i chose A becuase it seemed kind of right to me. I just wasn't really sure on this because i couldnt really pick anything out of the passage to help me. Are we just supposed to know this stuff? Can someone help me out and go through each answer and explain why its WRONG?

#115 in BS: It was hypothesized that the decrease in blood flow to the skin reulsted from a change in the activity of the sympathetic nerves to the skin. Which supports this hypothesis?

The answer is A- a change in norep. content of blood draining from the skin. I understand why this could work, but i chose C- a lack of epinep. receptors in skin blood vessels. .....and now that i just typed that out, is C wrong becuase they were talking about norepinepherine and not epinepherine?? and if not...then why is C wrong?

Thanks a lot!
 
For the verbal question #69,
A line from the passage was: "An outsider, I never understood the sunflower's golden language," which clearly implies that the author believes in choice A, that we cannot understand some intraspecies messages.
 
For #115 BS:
No you weren't wrong because of a norepinephrine/epinephrine distinction, because when it comes down to it both are a part of sympathetic nervous system.
I thought it was choice A because a sympathetic response is usually triggered by a change in hormone levels, not a change in the hormone receptors themselves. Any norepi/epi receptors in skin cells (and they do exist, or else vasoconstriction would not work) would not change. Hope that made sense..

I got #110 BS wrong too though...I thought it was choice C.
 
Can someone plese help me out with these? The explanations are so general and i feel like these are just things we should know, rather then obtain from the passage...

#110 in BS: Normally the immune system avoides attacking the tissues of its own body because:

A) special intracellular process recognizes only foreign antigens
B) the body does not make any antigens that the immune system could recognize
C) It changes its antibodies to be specific only to foreign antigens
D) it suppresses cells specific to the body's own antigens

---The answer is D but i chose A becuase it seemed kind of right to me. I just wasn't really sure on this because i couldnt really pick anything out of the passage to help me. Are we just supposed to know this stuff? Can someone help me out and go through each answer and explain why its WRONG?

#115 in BS: It was hypothesized that the decrease in blood flow to the skin reulsted from a change in the activity of the sympathetic nerves to the skin. Which supports this hypothesis?

The answer is A- a change in norep. content of blood draining from the skin. I understand why this could work, but i chose C- a lack of epinep. receptors in skin blood vessels. .....and now that i just typed that out, is C wrong becuase they were talking about norepinepherine and not epinepherine?? and if not...then why is C wrong?

Thanks a lot!
#110: lets focus on B cells to keep things simple (T cells have a different mechanism but the same result). The antigen specific method of B cells works because these cells essentially have a random aspect to the gene which produces antibodies, thus, the B cell can produce any of a multitude of different possible antibodies (with different light/heavy chains, etc.) Its entirely possible that some B cells would end up with antibodies that are sensitive to certain cells in our body, which i'll call body antigens. For this reason, all immune cells are matured in the thyroid. the thyroid essentially selects for cells which are insensitive to our body antigens. In the thyroid, cells which bind to our body antigens are killed off while those which are insensitive live on. With thus understanding, you can see that:
A is wrong, b/c its the development in the thryoid which kills off cells that recognize body antigens, not any intracellular activity. im guessing what they were getting at was the MHCI/II stuff, in which cells will essentially "chop up" certain peptide chains and "throw them up" to be analyzed by a T-cell. If the T-cell recognizes an antigen it becomes activated. Its entirely possible for a T-cell to become activated by body antigens in this way, however T-cells which would do this are killed off in the thyroid before they mature.
B is wrong, b/c the body does make antigens which could be recognized by the immune system, its just that the cells which recognize body antigens are killed off in the thyroid
C is wrong, b/c the antibodies are never changed, any given B cell produces only one type of antibody, period.
D is correct, it essentially summarizes the stuff going on in the thryoid which i mentioned above.

#115: the reason why C is wrong has nothing to do with the fact that its epinephrine vs. norep. notice what it says: a LACK of ep. receptors in skin blood vessels. if there is a lack of recepters, it means that the vessels are insensitive to epinephrine, which is a neurotransmitter which the symp. nervous system uses. how could the SNS innervate a tissue which is insensitive to its transmitter? Although it might use other means, such as Norep., one thing is for sure - the answer certainly doesnt SUPPORT the hypothesis, its either neutral or negative for the hypothesis.
 
  • Like
Reactions: 2 users
Can someone plese help me out with these? The explanations are so general and i feel like these are just things we should know, rather then obtain from the passage...

#110 in BS: Normally the immune system avoides attacking the tissues of its own body because:

A) special intracellular process recognizes only foreign antigens
B) the body does not make any antigens that the immune system could recognize
C) It changes its antibodies to be specific only to foreign antigens
D) it suppresses cells specific to the body's own antigens

---The answer is D but i chose A becuase it seemed kind of right to me. I just wasn't really sure on this because i couldnt really pick anything out of the passage to help me. Are we just supposed to know this stuff? Can someone help me out and go through each answer and explain why its WRONG?

#115 in BS: It was hypothesized that the decrease in blood flow to the skin reulsted from a change in the activity of the sympathetic nerves to the skin. Which supports this hypothesis?

The answer is A- a change in norep. content of blood draining from the skin. I understand why this could work, but i chose C- a lack of epinep. receptors in skin blood vessels. .....and now that i just typed that out, is C wrong becuase they were talking about norepinepherine and not epinepherine?? and if not...then why is C wrong?

Thanks a lot!
#110: lets focus on B cells to keep things simple (T cells have a different mechanism but the same result). The antigen specific method of B cells works because these cells essentially have a random aspect to the gene which produces antibodies, thus, the B cell can produce any of a multitude of different possible antibodies (with different light/heavy chains, etc.) Its entirely possible that some B cells would end up with antibodies that are sensitive to certain cells in our body, which i'll call body antigens. For this reason, all immune cells are matured in the thyroid. the thyroid essentially selects for cells which are insensitive to our body antigens. In the thyroid, cells which bind to our body antigens are killed off while those which are insensitive live on. With thus understanding, you can see that:
A is wrong, b/c its the development in the thryoid which kills off cells that recognize body antigens, not any intracellular activity. im guessing what they were getting at was the MHCI/II stuff, in which cells will essentially "chop up" certain peptide chains and "throw them up" to be analyzed by a T-cell. If the T-cell recognizes an antigen it becomes activated. Its entirely possible for a T-cell to become activated by body antigens in this way, however T-cells which would do this are killed off in the thyroid before they mature.
B is wrong, b/c the body does make antigens which could be recognized by the immune system, its just that the cells which recognize body antigens are killed off in the thyroid
C is wrong, b/c the antibodies are never changed, any given B cell produces only one type of antibody, period.
D is correct, it essentially summarizes the stuff going on in the thryoid which i mentioned above.

#115: the reason why C is wrong has nothing to do with the fact that its epinephrine vs. norep. notice what it says: a LACK of ep. receptors in skin blood vessels. if there is a lack of recepters, it means that the vessels are insensitive to epinephrine, which is a neurotransmitter which the symp. nervous system uses. how could the SNS innervate a tissue which is insensitive to its transmitter? Although it might use other means, such as Norep., one thing is for sure - the answer certainly doesnt SUPPORT the hypothesis, its either neutral or negative for the hypothesis.
 
Hi,
Alright I'm having a really tough time figuring out the logic behind the verbal q's for the "atonality" passage and the "low/middle/high road" passage. The answer explanations aren't helping much either...they tell me why the right answer is right, but they don't tell me why another answer isn't right...which, considering the ambiguity of some verbal q's, isn't helpful at all.

Ugh I can't exactly post all the q's from those two passages (though help on any is appreciated) but here are some I struggled with most:

Passage IV (Atonality)

That Schoenberg rejected "general principles regulating simultaneity and progression" means that he rejected:
a) chaotic chord progression
b) broadened range of tonal relationships
c) chromatic scale
d) standard system of movement from certain tonalities to other prescribed tonalities
I thought it would be C, because Schoenberg's idea was a departure from the traditional chromatic scale...I see that D is plausible, but not why C is wrong...Did he reject the chromatic scale or only free it from its 'diatonic associations'?

According to the passage, the 12 tone scale rejects systems based largely on traditional relationships between notes. One can most reasonably conclude that:
a) diatonic scales are conventions composers may accept/reject
b) traditional scale is needlessly repetitive
c) traditional and 12 tone systems are more similar than different
d) 12 tone composers have not been able to equal quality of traditional compositions
I chose C on this one...and I still don't get why that's wrong or why A is right.

Passage V (low/middle/high road)
A recently founded small firm that follows the high road described in the passage is considering selling stock shares to public. Author would advise against this step because:
a) investors have no reason to care about quality of the firm's products
b) investors are likely to doubt the wisdom of the firm's personnel policies
c) firm is unlikely to produce the quick profits investors want
d) firm is unlikely to attain a competitive status in the market
I chose B because it had at least an inkling of the author's idea/stance (reference to personnel policies). Although C sounds plausible, I don't see how it follows directly from anything the author said.

Thanks! Any help appreciated...7R verbal has been brutal :confused:
 
I did the CBT only so I didn't have the atonality passage, but in response the the high-road question:

a) This is one of those freebie answers that they throw in there that can be pretty much eliminated immediately. There is no reason to think that investors have no reason to care about the quality of the firms products, and it is not suggested in the passage.

b) The author would not advise against stock based on the fact that investors are likely to doubt the wisdom of the firm's personnel policies. There is no discussion of stock in the article (that I recall), but this goes completely against what the author has been arguing for the entire article. The author believes the high-road policy is wise and would not advise against stock based on this principal.

c) The high-road policy involves a lot of training and substantial funds on the part of the firm itself. This would lead to a long period where there is little to no profit for the firm. This seems to be the most logical choice

d) Again this choice does not jive with the authors argument throughout the passage. The author would argue that the firm is more likely to attain a competitive status in the market due to the high-road policy.


So in the end this question is less about the investor and more about what the author has said throughout the passage (as are most VR questions with respect to some newly introduced 3rd party). A,B, and D can be eliminated based solely on the authors argument throughout the passage, and you can chose C even if you can't reason out the fact that the high-road firm would logically produce little to no profit initially with its extensive employee training program gobbling up funds and creating a lot of overhead.
 
Also, everyting Kish said is right except for it is the THYMUS where lymphocytes mature, not the thyroid.
 
I wanted to ask about Q44 on 7CBT.

Question asks:

Because earthquakes were triggered by what ultimately must have been comparitively minor energy transfers from the distant Landers quake, the triggered quakes probably occurred where:

A) significant local stress forces the earth's crust were already in precarious equilbrium

B) the earth's crust was subjected to reinforcing resonant effects that cumulatively built up local stress


I left out C & D because they were absolutely wrong.

Now I was wondering what made A a better answer than B. Even the answer admits that B sounds like a good answer.

My problem with A being the answer is that the passage states that at a distance 4L the deformations are less than the daily distortions due to tidal forces.

Now, stresses are never mentioned, but I was thinking that if at 4L away the forces are already small enough such that they deform the earth as much as normal forces do. Yet triggered quakes were reported to occur up to 17L away. That would mean the stress/forces in question were tiny??

Does anyone have a definitive answer to this question - and any tips on how to handle these sort of "nonphysics" questions on the actual test??

THANKS!
 
Does anyone understand dimensional analysis for #51 in PS? I got it right buy guessing in multiples of 3, but I'm not sure how you would get it otherwise
(Appx how many moles Al+3 are reduced when 0.1 faraday of charge passes through a cell during the production of Al?)
:confused:
 
Does anyone understand dimensional analysis for #51 in PS? I got it right buy guessing in multiples of 3, but I'm not sure how you would get it otherwise
(Appx how many moles Al+3 are reduced when 0.1 faraday of charge passes through a cell during the production of Al?)
:confused:

Isn't one Faraday equal to the charge on one mole of electrons? We need to add three electrons to Al+3 to reduce it to Al. Given 0.1 Faraday of charge, we could reduce 0.1 moles of something like Al+3 to Al. Since we need three times as many electrons to reduce the Al+3 all the way, you can only reduce 1/3 as much: (1/3) x 0.1 = 0.033.

Of course, I didn't realize this during the test. Instead I used this method:

Hmm...all the answers are less than 1. What's the easiest way I could get an answer less than 1 with the numbers given? Ah, 0.1/3 = 0.033...good enough for me.
 
  • Like
Reactions: 1 user
The Doppler shift can impact any wave. In this case, the wave is an impact wave travelling through the ground away from the rupture. The source of the wave is the rupture, and given that the rupture is moving, the wave it emits will be Doppler shifted. You can thus treat it as a source moving north emitting a wave, much like a car travelling north emitting a sound.

The explanation from the BR answer sheet is:
  • The primary rupture is moving from south to north, so the Doppler effect is associated with north-south movement and not east-west movement. This eliminates choices A and B. Seismic waves propagating from this rupture are Doppler shifted by the motion of the rupture. Seismic waves emitted to the north are compressed by the motion of the rupture while seismic waves moving south are elongated by the motion of the rupture. Compression results in a shorter wavelength, so the wavelength of seismic waves moving to the north is decreased while the wavelength of seismic waves moving to the south is increased. The best answer is choice C.


If the rupture is acting like a source wouldn't the wave that is coming towards it (south) decrease and the one going away (north) increases??
 
um did anyone think BS was a bit on the tougher side on this one?
 
um did anyone think BS was a bit on the tougher side on this one?

It did feel a bit tougher

I thought the verbal on this one was weird. Anyone else agree? I went from a 12 in VR on AAMC 6 to a 10 on this one.

The passage about nature and the flowers and the insects was a bit strange, I think its the first one like that I have come across on the AAMCs
 
Wow, I'm in the middle of this test right now - break between VR and BS - and I thought the VR was unreal. I've never seen passages that long in AAMC VR, those were as long as some of the absurd Kaplan ones. Made me realize I need to adjust my methods a little just incase this happens on test day. I also thought the PS was very challenging ... a ton of reading passages in it too. AAMC 6 and AAMC 7 seem so weird to me. But god that VR sucked.
 
Could someone post or send me a scoring chart for this exam? I want to know the number of correct responses needed to move up in score in my sections :p

Thanks~
 
hey can someone please explain how we know its SN1 or SN2 reaction here? is this just a basic knowledge we should know? also, in kaplan explanation, it says we realize its SN1 by Nucleophile, Cl, which is weak. I thought strong bases such as Cl is good nucleophile? thanks
 
Can someone explain to me the reasoning behind the following questions/answers? I am so confused, I got four questions wrong on the Nematode passage (VII). Thanks

#138 The results from experiment 2 indicate that the signaling interaction of the two-cell stage most involves which class of molecules? I don't understand why can't it be rRNA??

#139 The results from experiment 3 indicate that gut specification during the four stage cell requires which cell to cell communication? I have no idea how to come up with the answer :confused:
 
hey can someone please explain how we know its SN1 or SN2 reaction here? is this just a basic knowledge we should know? also, in kaplan explanation, it says we realize its SN1 by Nucleophile, Cl, which is weak. I thought strong bases such as Cl is good nucleophile? thanks


Cl- (which is what you will have in this problem after the first rxn step, which is an acid-base rxn between the alcohol and the HCl) is not a stellar nucleophile for a couple of reasons: 1) Cl is quite electronegative and so is "happy" with a negative charge, and 2) Cl is (relatively) large and so is able to accommodate a negative charge pretty well. these same reasons explain why Cl is such a good leaving group.
 
omg i just took this (full length version), 15 ps 11 vr 15 bs, I thought the verbal was RIDICULOUS, I missed 12! but somehow that translates into an 11. Glad to see everyone else thinks the same way though, I was worried for a bit.


I missed 3 on the verbal for a score of 13. that seems to be a weirdly large gap between an 11 and a 13 (9 more wrong answers only knocks your score down two points?)
 
The base in Step 5 of the synthesis abstracts a proton from which of the labeled (a-d) carbon atoms of Compound 7 (Figure 2)?

The answer is A. I picked C.

I understand why A and C hydrogens are acidic, but isn't hydrogen A also an acidic hydrogen? It is attached to the alpha carbon of a ether, which is also electron withdrawing.

Thanks for the help!
 
hey can someone please explain how we know its SN1 or SN2 reaction here? is this just a basic knowledge we should know? also, in kaplan explanation, it says we realize its SN1 by Nucleophile, Cl, which is weak. I thought strong bases such as Cl is good nucleophile? thanks

I'm trying to reason this one out myself so correct me if I'm wrong. The way we can tell if it is an SN1 or SN2 reaction is:
SN2 is favored by polar aprotic (non-h bonding) solvent.
SN1 is favored by protic solvents.

It is SN1 because alcohol is protic. To favor an SN2 mechanism, protic solvents such as alcohols should be avoided.

Cl is also a good leaving group.
 
The base in Step 5 of the synthesis abstracts a proton from which of the labeled (a-d) carbon atoms of Compound 7 (Figure 2)?

The answer is A. I picked C.

I understand why A and C hydrogens are acidic, but isn't hydrogen A also an acidic hydrogen? It is attached to the alpha carbon of a ether, which is also electron withdrawing.

Thanks for the help!

The answer is C. so if you picked C, you are correct.
 
I'm trying to reason this one out myself so correct me if I'm wrong. The way we can tell if it is an SN1 or SN2 reaction is:
SN2 is favored by polar aprotic (non-h bonding) solvent.
SN1 is favored by protic solvents.

It is SN1 because alcohol is protic. To favor an SN2 mechanism, protic solvents such as alcohols should be avoided.

Cl is also a good leaving group.

your thinking about solvents and SN1/SN2 is correct, but this question says nothing about the solvent used for this reaction. here, the correct answer can be ascertained by going through the answer choices and classifying the alcohol in terms of degree of substitution. two of the answer choices are primary alcohols, one is a secondary alcohol and one (the correct choice) is a tertiary alcohol. the question asks which alkyl halide is "most readily" prepared...this means, which reaction is fastest. the most substituted (ie., tertiary) alcohol forms the most stable carbocation when the leaving group leaves, therefore that is the one that will react the fastest
 
BS Passage VI (H. pylori)

Which of the following statements explains most plausibly why host antibodies are ineffective against H. pylori?

Answer: Antibody proteins may be denatured in the harsh environment of the stomach

What I picked: H. pylori infection may suppress the activity of the immune system.

The correct answer does make sense but the passage clearly says:

"Although many individuals develop antibodies against H. pylori antigens, these antibodies rarely eradicate the infection; evidently, this pathogen has developed effective ways to elude host defenses.

Don't tell me the pathogen has some tricky way to avoid host defenses and then make a correct answer "Oh... never mind.. actually it was just the acidic nature of the pH and has nothing to do with actions by H. pylori like we just said in the last sentence of the passage."

(-_-)
 
BS Passage VI (H. pylori)

Which of the following statements explains most plausibly why host antibodies are ineffective against H. pylori?

Answer: Antibody proteins may be denatured in the harsh environment of the stomach

What I picked: H. pylori infection may suppress the activity of the immune system.

The correct answer does make sense but the passage clearly says:

"Although many individuals develop antibodies against H. pylori antigens, these antibodies rarely eradicate the infection; evidently, this pathogen has developed effective ways to elude host defenses.

Don't tell me the pathogen has some tricky way to avoid host defenses and then make a correct answer "Oh... never mind.. actually it was just the acidic nature of the pH and has nothing to do with actions by H. pylori like we just said in the last sentence of the passage."

(-_-)
I think there might be a distinction between "eluding" the immune system and "suppressing" it. For example, "elude" might imply that it hides its antigens in such a way that it avoids being presented by MHC class I molecules. To me, "suppress" sounds more like shutting down the plasma cells so that they can't produce antibodies, which isn't suggested in the passage. I agree that the last sentence is very misleading, though.
 
I think there might be a distinction between "eluding" the immune system and "suppressing" it. For example, "elude" might imply that it hides its antigens in such a way that it avoids being presented by MHC class I molecules. To me, "suppress" sounds more like shutting down the plasma cells so that they can't produce antibodies, which isn't suggested in the passage. I agree that the last sentence is very misleading, though.

Right.

That makes sense but I still never would have picked the stomach answer since the passage seemed to indicate the 'eluding' was a direct result of something the bacterium contains/has the ability to do, not something that the host does. So I just picked the suppress answer not because I thought it was right but because it seemed to be the best of the choices presented.

Ah well. They should have just left that last sentence out!
 
Isn't one Faraday equal to the charge on one mole of electrons? We need to add three electrons to Al+3 to reduce it to Al. Given 0.1 Faraday of charge, we could reduce 0.1 moles of something like Al+3 to Al. Since we need three times as many electrons to reduce the Al+3 all the way, you can only reduce 1/3 as much: (1/3) x 0.1 = 0.033.

Of course, I didn't realize this during the test. Instead I used this method:

Hmm...all the answers are less than 1. What's the easiest way I could get an answer less than 1 with the numbers given? Ah, 0.1/3 = 0.033...good enough for me.

This problem is really bothering me...will someone help me out. I got the problem right because of the answers choices but I don't understand why you divide .1 by 3.

Since it's .1 change per mole, I thought you would multiply .1 times 3 to get the number of moles.
 
This problem is really bothering me...will someone help me out. I got the problem right because of the answers choices but I don't understand why you divide .1 by 3.

Since it's .1 change per mole, I thought you would multiply .1 times 3 to get the number of moles.


It has been a couple of weeks, so I don't recall the problem exactly.

But isn't it asking how many moles of Al are required? Since each mole of Al would provide 3 electrons, you would divide by 3. Something like that?
 
Approximately how many moles of Al3+ are reduced when 0.1 faraday of charge passes through a cell during the production of Al? (Note: Assume there is excess Al3+ available and that Al3+ is reduced to Al metal only.)
 
BS Passage VI (H. pylori)

Which of the following statements explains most plausibly why host antibodies are ineffective against H. pylori?

Answer: Antibody proteins may be denatured in the harsh environment of the stomach

What I picked: H. pylori infection may suppress the activity of the immune system.

The correct answer does make sense but the passage clearly says:

"Although many individuals develop antibodies against H. pylori antigens, these antibodies rarely eradicate the infection; evidently, this pathogen has developed effective ways to elude host defenses.

Don't tell me the pathogen has some tricky way to avoid host defenses and then make a correct answer "Oh... never mind.. actually it was just the acidic nature of the pH and has nothing to do with actions by H. pylori like we just said in the last sentence of the passage."

(-_-)

This question was absolutely horrible. There are no questions about it. I think the only way to answer it correctly is know what the tester wanted.

1. antibodies are rarely effective against bacteria. It is a reason why vaccines are ineffective against bacterial pathogens. This is my field of research and this comes up quite often. This is much more true about gram + pathogens like Staph & Strep, which is a partial reason for their increased virulence. I didn't know that Heliobacter was a gram negative, so I chose this one. Furthermore, it is an ambiguous answer. Effective how? against curing infection--that would be next to nill? Preventing inoculation or beginning of infection? Somewhat... This is a huge case where being an "expert" in the field the question pertains to can completely give you the shaft.

2. Abs are notoriously easily to denature & otherwise f-up. However, there are Abs that can be active at lower Ph's, we require some for specific experiments.

3. The passage is completely misleading, as you said.

4. Your answer is right, except for suppress. Bacteria don't really suppress the immune system, they are just dam good at evading it.

Horrible question, I would have never picked the "right" answer, and I don't know how to change my knowledge base so that it even comes close to fitting.
 
Approximately how many moles of Al3+ are reduced when 0.1 faraday of charge passes through a cell during the production of Al? (Note: Assume there is excess Al3+ available and that Al3+ is reduced to Al metal only.)

To reduce Al+3 to Al, you need 3 electron per Al+3.
Since 0.1 faraday represents charge carried by 0.1 moles of electron, 0.1/3 moles of Al+3 will be reduced.
 
I really appreciate you in taking the time to help me undestand this...but it's still not clicking in to me and its killing me! :scared:

This is my thought process with what you said

.1 C /1 mol X 3 mol electron = 3 C


Or is it that my units are messed up? 3 mol electron should be 3 C of charge so that you would divide and get moles of Al?
 
I really appreciate you in taking the time to help me undestand this...but it's still not clicking in to me and its killing me! :scared:

This is my thought process with what you said

.1 C /1 mol X 3 mol electron = 3 C


Or is it that my units are messed up? 3 mol electron should be 3 C of charge so that you would divide and get moles of Al?

I think you are misunderstanding the question.

The problem is NOT asking for change in charge when Al+3 is reduced. Instead, it's asking how much Al+3 will be reduced by 0.1 Faraday. You are supposed to take "external" electrons available and add them to Al+3.

Suppose you have 3 electrons available. You are trying to reduce Al+3. You can only reduce one Al+3 ion to Al.
 
Top